Physics for Scientists and Engineers: A Strategic Approach with Modern Physics (3rd Edition)

Published by Pearson
ISBN 10: 0321740904
ISBN 13: 978-0-32174-090-8

Chapter 33 - Electromagnetic Induction - Exercises and Problems - Page 997: 14

Answer

${\bf+4.69}\;\rm T/s$

Work Step by Step

We have here a loop and a changing flux, so we must have an induced current. We know that the induced current is given by $$I=\dfrac{\varepsilon}{R}\tag 1$$ where the induced emf $\varepsilon$ is given by $$\varepsilon=\left|\dfrac{d\Phi}{dt}\right|$$ where $\phi =AB\cos\theta$ and here $A$ is constant and $\theta= 0^\circ$, so that $$\varepsilon=A\left|\dfrac{dB}{dt}\right|$$ where here $A=L^2$ $$\varepsilon=L^2\left|\dfrac{ dB}{dt}\right| $$ Hence, the rate of the magnetic field strength changing is given by $$\dfrac{dB}{dt}=\dfrac{\varepsilon}{L^2}$$ Plug $\varepsilon$ from (1); $$\dfrac{dB}{dt}=\dfrac{IR}{L^2}$$ Plug the known, from the given graph, $$\dfrac{dB}{dt}=\dfrac{(150\times 10^{-3})(0.20)}{(0.08)^2}$$ $$\dfrac{dB}{dt}=\color{red}{\bf4.69}\;\rm T/s$$ As we see from the given figure, the induced current creates a magnetic field with a direction out of the page while the external magnetic field is into the page. We know that the induced flux is opposing the original flux, so the magnetic field increases.
Update this answer!

You can help us out by revising, improving and updating this answer.

Update this answer

After you claim an answer you’ll have 24 hours to send in a draft. An editor will review the submission and either publish your submission or provide feedback.